Hello,
I chose E and can't really understand why it is wrong whereas C is correct. Can someone please break down the logic or reasoning behind this argument? Thanks in advance
110 posts in the last 30 days
Hello,
I chose E and can't really understand why it is wrong whereas C is correct. Can someone please break down the logic or reasoning behind this argument? Thanks in advance
Really struggled with this on the exam, and I still missed it during BR. Specifically, I had a really tough time differentiating answers A, B, C, and D. Can someone help me out with those answer choices? Here is my breakdown:
TV show MOST depend on ad funding
TV show MOST cancelled with no ad funding
Ad funding----->Some people watching buy the advertised products
Most people don't buy the advertised products------>Shows cancelled soon
Thus, person who thinks a show is worth preserving------>should buy the stuff advertised during the show.
What I am looking for: I need something that concludes that people should buy the advertised stuff.
Answer A: This is what I chose, but it doesn't fit the facts of the passage. All we know is that MOST TV shows would be cancelled with no ad funding, and that if MOST people don't buy the products, the show sill be cancelled. This answer choice is too certain about the inevitability of cancellation.
Answer B: I don't understand how this answer choice is that different from A. How do we know about the certainty of cancellation?
Answer C: We don't know if the TV show is ACTUALLY worth preserving, only that someone "feels"/believes that it is.
Answer D: What makes this one incorrect? If B is correct, how is this one incorrect?
Answer E: "Feel most strongly?" This was the only one I could confidently eliminate since we don't care about the degree of caring.
How is A a sufficient assumption? I didn't like any of the answer choices, so I pretty much guessed on this one. I thought A was the least attractive answer choice because it lacks the conditional nature that is typical for sufficient assumptions. My understanding of the argument is this:
We can't figure out how effective a certain model cleans simply be looking at how powerful the motor is. This is because the efficiency varies a lot, even with identical motors.
The sufficient assumption I was looking for was this: if efficiency varies (even with identical motor power), then we can't determine how effective the model cleans.
How does A paraphrase this?
I don't understand how B is the answer. In the lessons regarding weakening, I was under the impression that we were to look for ways to weaken the connection between the Premises and the Conclusion. If the question states that "many human diseases are genetically based," then how does B not just attack the Premises? I mapped out ... human diseases (HD) are genetically based (GB)... HD -> GB. Then some of Cat's (CG) genetics are the same to humans (HG). ... CG(--s--) HG. Then I said Some of Primates (PG) genetics are the same as Humans .... PG (--s--) HG... the conclusion is humans have many diseases (HD) in common with Cats (CD) ...HD --m-->CD
I figured (albeit I now see that it was an incorrect conclusion... I just dont know why) that the question was trying to jump from some cats and some primates genes are the same as humans so therefore Humans have diseases in common with Cats. Answer C could provide a weaker link with the premise/conclusion connection by pointing out that a some correlation does not imply a most correlation.
Any help or guidance would be appreciated.
When I read the question I assumed that the debris hurled into the atmosphere causing the blocking of the sun and extinction of the dinosaurs was only in the Yucatan Peninsula. Which is why answer choice B was attractive and I picked it rather than answer choice E. I understand why E is correct, but can somebody explain why or how I should have assumed the debris from the asteroid was around the world versus just affecting the Yucatan Peninsula?
Anyone from the Chicago area interested in getting a group together for mock LSAT exam?
Did anyone else find these games sections more difficult than usual? I am a pretty consistent -0 to -2 on the games sections, but damn the A, B, and C games sections are brutal. For example, games 3 and 4 on PT A each ate up like 12-13 minutes. I will definitely be redoing these sections in a few weeks. Did anyone else feel the same way about these games?
1. People with long legs make good runners. Everyone in Ashley's family has long legs. Therefore, Ashley would make a good runner. (I think it's a bad argument because just because Ashley's family has long legs doesn't necessarily mean she does, right?)
2.In Los Angeles everyone over the age of 18 who drinks also smoke. But not everyone in Los Angeles over the age of 18 who smokes are drinks. It follows tear among people over the age of 18 in Los Angeles there are more who drink than there are who smoke. (this felt like a bad argument to me)
3. All of the painting in the Janet Collection will be put up for auction next week. Since the paintings to be auctioned next week are by a wide variety if artists, it follows that the paintings in the Janet collection are by a wide variety of artists (This felt like a bad argument to me)
4. A writer's first book will become a best-seller only if it has a romantic setting and a suspenseful plot. Since many author's first novels have neither, it follows that not many first novels become best-sellers. (I thought this was a good argument)
5. Some short poems are thematically pluralistic, since some sonnets are characterized by such pluralistic, and all sonnets are short poems. (I thought this was a bad argument, because I couldn't diagram it)
6. Most of the people in Los Angeles buys gasoline on Mondays only. But almost everyone in Los Angeles buys groceries on Tuesday only. It follows that fewer than half of the people in Los Angeles buy gasoline on the same day on which they buy groceries.
Hello,
I don't understand why D is the correct answer. I chose C. Could someone please explain why the right answer is right and the wrong answers are wrong? Thank you
Can someone explain how A is the correct answer? I got this question correct by POE, but during BR, I just can't figure out how A is explicitly correct.
Stats guy: Changes in the sun’s brightness correlate with land temperatures on Earth. Clearly, and contrary to what meteorologists think, the sun’s brightness is the main cause of land temperature.
Meteorologist: You are wrong, dude! Any professional meteorologist will tell you that climate is really complicated. There is no significant part that is controlled by one thing.
What I am looking for: The stats guy makes the typical causation/correlation flaw. Plus, even if the sun were to be a causal factor, the stats guy hasn’t given any evidence that the sun is the MAIN cause. The meteorologist is making an appeal to professionals, but it is actually pretty weird that he is doing this. The stats guy says that meteorologists can’t be trusted, so the meteorologist citing other meteorologists won’t do anything to convince the stat guy. I was expecting that the correct answer was going to talk about this idea (an irrelevant appeal to authority).
Answer A: This is it simply by POE. That’s really all I got because I don’t see the “specific case” nor the “invoking of a relevant generalization” in the meteorologist’s retort.
Answer B: What single counterexample? What generalization is false?
Answer C: I think this must be false. The meteorologist seems to be arguing the opposite of this idea: there is no single cause because climate is very complicated.
Answer D: Experimentally tested? He doesn’t bring this idea up.
Answer E: What unfavorable evidence? Systematically neglected? This just isn’t done.
Hi, I was looking at PT27 and found out that question 17 in Game 3 seemed quite odd. The answer is "E" here but I did not understand why it is so. The question asks " which of the following must be true" and "E" i think is only "partially" correct... Although it is true that G and L gets to see different films, L and M COULD see the same film. Thus, it's wrong to say that G, L, and M do not all see the same film.
Any thoughts here??? Help me!
A tall tulip is not a tall plant. (eg. compare a tulip with an apple tree or pine)
no (only tall tulips) does not equal (only tall plants.)
It is not necessary to know for the question, but gorillas are very large, three times or more the size of a human, monkey or chimpanzee.
(C is correct). All the Gorillas are small, but it is flawed to saw that they would all be small primates.
[only small gorillas] does not equal [only small primates]
even a small gorilla is massive compared to a lemur or monkey.
E is the answer.
This conclusion is: CPUE number same -> shark numbers are same
To weakedn the conclusion, we need to establish that the way to reach CPUE somehow has changed since 1973.
This question defines the CPUE carefully.
E is the only answer that could address catches 'per hour' and the other answers are not relevant.
E: (Technology improvement -> sharks still easy to find EVEN THOUGH numbers have dropped -> maybe shark numbers not equal)
I don't see how D is incorrect. E is obviously correct, but how is D wrong?
Professor V's book is getting people riled up over the origins of glassblowing (really exciting stuff). If V is right, then there would be not enough evidence to conclude that glassblowing started in Egypt (which is what most historians think). This traditional view is still accepted by the majority of historians. If V is right, we must conclude that glassblowing started elsewhere.
Answer A: So what? This doesn't even describe a type of flaw to begin with.
Answer B: The argument isn't circular.
Answer C: We don't need the criteria.
Answer D: Doesn't the argument do this? Isn't what "most historians" claim (line 6) conflated with the "traditional view" in line 8? How is this not the flaw?
Answer E: This is definitely the flaw of the argument, but I can't figure out how it is better than D.
During the exam, A just seemed right, but during BR, I am having a ton of time justifying it. I'd also like some further analysis on answer C.
The government increased the minimum wage. The minimum wage increases the museum's operating expenses, which now are less than the revenues (so the museum is profitable). Thus, the museum will either raise admission fees or decrease services; these hurt the museum going people.
What I am looking for: I couldn't think of a good necessary assumption. All I could think of was raising admission fees or decreasing services is bad for museum goers. I kept thinking to myself "no shit."
Answer A: I was turned off by this due to the word "significantly," but the other answer choices were clearly incorrect. If it is not true that some of the museum's employees are not paid significantly more than the minimum wage, then what does this mean? I take it to mean that the employees are either paid slightly higher than minimum wage, at minimum wage, or below minimum wage. How does this actually wreck the argument?
Answer B: What if the revenue fluctuated? It doesn't matter.
Answer C: I am sure some are, but so what? If you negate this: no one is paid more than the current minimum wage, then this might strengthen the argument since it would suggest that the increase in the minimum wage is going to impact the operating expenses further.
Answer D: So what about the annual number of visitors?
Answer E: So what?
EDIT: I initially mistyped the part of the summary dealing with profit.
Can someone please be so kind to read my notes from the particular lessons listed below? I do not know which definitions are correct. Different lessons are contradicting each other. Please help!
Logical Reasoning (Lesson 25 of 40, 9m)
Or (3 options) = A is selected, B is selected or both A and B are selected.
Logical Reasoning (Lesson 27 of 40, 5m)
Not both – negate the Necessary, then contropose. In short, 1, A or B must be selected.
Advanced Logical Reasoning (Lesson 2 of 15, 5m)
Or is different because it has (2 options) = A is selected or B is selected. A and B are not selected together.
In-Out Games (Lesson 1 of 20, 5m)
Or is back to the original lesson, (3 options) = A is selected, B is selected or both A and B are selected.
Not both is different because 1, A or B can be is selected or nothing has to be selected.
HI All,
I've been studying since for almost a year now for the december lsat. Completed the trainer and 7sage course thoroughly. Around august i started feeling a little worn out so took a break for 3 weeks. Now I am back and cranking on those Practice tests. My reason for concern is that just before i left my prep i was scoring 164-168 and 170-172 BR, however, now i started of with 154 actual and 160 BR. Should i be worried?
I studied on my own while working a full-time job using the PowerScore Bibles, and I felt fairly confident. I had been doing well on PTs (around 160), but the night before the test I had a lot of unexpected things happen in my personal life and I was definitely distraught the morning of the October 2015 test. I just got my scores back yesterday and I made a 147. So now I'm going to try a different mode of study, 7Sage, and retake the LSAT in February. Any advice or tips? I would be thrilled with a 165, but now that I've done so poorly on the test I'm afraid of making the same mistakes twice.
I really don't understand how the correct answer choice addresses the paradox.
After some people witnessed a fake crime, two lawyers questioned the witnesses. The first lawyer tried top mess them up and the second lawyer tried to correct the inaccuracies. The witnesses who gave testimony that had FEWER inaccuracies during lawyer one's interrogation had the greatest NUMBER of inaccuracies than most of the other witnesses when questioned by lawyer two.
What I am looking for: I was convinced during the exam that the correct answer choice was going to be some bias the accurate witnesses had over the other witnesses. Or, that the second lawyer phrased questions that confused the accurate witnesses/asked questions about different topics than lawyer one. These were not answer choices, so I ended up spinning my wheels a lot on this.
Answer A: This explains possibly why they were more accurate at the start, but why did they have more inaccuracies later?
Answer B: This is a lot like A. If they had better memory, why did they all of a sudden have inaccurate testimony? This makes it weirder, I think.
Answer C: I don't see how this helps resolve the problem. I eliminated it during the exam and during BR since I thought it made the problem even weirder. I think it does explains why they were so accurate to begin with: they were not swayed by lawyer one's bullshit. But how does it explain the fact that they were more inaccurate later? To get this to work, don't you need to assume that either these witnesses became less confident after lawyer one's questioning (which seems bizarre since this sort of contradicts this answer choice's entire point) or that the second lawyer's line of questioning focused on the stuff that these witnesses didn't remember? But how do we assume that this latter case is true? Additionally, wouldn't the "most of the other witnesses" also be inclined to increase their inaccuracy as well? I just don't understand how this makes the paradox anything but weirder, let alone resolve it.
Answer D: This definitely makes the paradox weirder. If they were unsure to begin with, then how were they so accurate?
Answer E: This is what I chose during the exam/BR since it was POE. I didn't really understand how it actually resolved the paradox (which was flag one/the reason why I marked it for BR). I think to get this to work, you have to assume that they gave so many more details, that they got a lot of stuff accurate since they were spewing details out left and right (this would explain the accuracy part). To resolve the other part, you have to assume that the "most other people" started remembering things more accurately, so those people improved over the originally accurate people. This was my line of reasoning during the exam and BR, but I knew I was making way too many assumptions. However, since I could at least think of a plausible reason for this one, I picked it.
I have spent about 30 minutes on this one, and I just don't see how A is correct or how C is incorrect.
Context: lines were crossed between the Jacksons and Sara.
Here was the sufficient assumption I was looking for; I used this to pick out the PSA:
Helpful to Sara and Not difficult for Jacksons to to pass on correct number----->Laudable action by Jacksons
Jacksons do not pass on correct number/Jacksons only tell caller that they dialed wrong number--->Action not wrong by Jacksons
Answer A: Helpful---->Laudable. Wrong----->Led person to believe . How is this the correct answer? Isn't the first condition missing the idea of something not being difficult? Also, -Led person to believe----> Not wrong? Where does the stimulus imply that this isn't wrong? To me, the passage doesn't link this idea.
Answer B: Completely wrong.
Answer C: This is what I chose. Helpful and Easy-----> Laudable and not wrong. Isn't this really good? Sure, easy isn't necessarily "not difficult," but this is a PSA question, not a SA question.
Answer D: Laudable--->Blah blah blah. Laudable should be the necessary condition.
Answer E: Laudable---> Blah blah blah. Same as D.
For the past couple weeks now I have been doing timed LR sections and I noticed that I have been going too fast, completing all 25 questions within the 35 minutes, leading to a score always in the -9 - -11 range. The only recommendation I ever got into how to remedy this problem was from my Princeton Review instructor, who told me to "slow down" (obviously).
So for my last 3 timed LR sections I have been doing just that. I've been guessing on 5 - 6 questions, yet I still get the same -9 - -11 score. I don't have any problem with the BR process - I just feel like my brain doesn't want to cooperate under timed conditions.
Does anyone have any advice on how I can improve and break past this?
Perfect LG section on Oct test (finally -- 3 tries!). Wouldn't have been possible without JY's awesome videos!
Did you guys all get your scores?
Can anyone explain the incorrect answer choice "B" because I'm having trouble getting through the convoluted way it's written?
I am still confused why the conclusion is adequate productivity --> high- tech technology. I negated the high tech technology part because of the "not" present in the sentence. I tried reviewing my notes and I can't find where he explains in the negation of conditional logic that this is viable.